Condición de equivalencia de Continuidad Absoluta

Permítanme reafirmar mi punto. La intuición detrás de esta construcción es sencilla. Dejar F Sea una función creciente continua. Saca pedazos de la gráfica de F que corresponde a una colección de subintervalos disjuntos finitos, luego pégalos para hacer una nueva función gramo . Si reclamamos cada gramo debe ser continuo, entonces por la definición de continuidad d ϵ tal que “la longitud total de los intervalos < ϵ ” implica “variación total de F en estos intervalos< d " lo que significa F es continuo absoluto. Esto no es tan sensato, así que planteo la hipótesis de que gramo no es necesariamente continua incluso si F es.


F : I R es continuo Tomar un número finito de subintervalos [ X 1 , y 1 ] [ X 2 , y 2 ] , . . . , [ X norte , y norte ] = k I , dónde k m ( [ X k , y k ] ) = k , es decir | k | = k .

Consideremos la parte del gráfico de F restringido en k . Sigue una definición rigurosa.

Definir función gramo : [ X 1 , X 1 + k ] R tal que, si X [ X 1 , y 1 ] , entonces gramo ( X ) = F ( X ) , si X [ y 1 , y 1 + y 2 X 2 ] , entonces gramo ( X ) = F ( X y 1 + X 2 ) + C 1 , dónde C 2 = F ( y 1 ) F ( X 2 ) . Así definimos X 1 = X 1 , y 1 = y 1 , X 2 = y 1 , y 2 = X 2 + y 2 X 2 y, recursivamente, X k = y k 1 , y k = X k + y k X k .

Si ( X + y k ) [ y k , y k + y k + 1 X k + 1 ] , entonces gramo ( X + y k ) = F ( X + y k ) + C k , y C k = gramo ( y k ) F ( y k ) .

Llamamos gramo un recorte de F en k :

gramo = C yo i pag ( F , k ) . Intuitivamente, gramo ( X ) es exactamente equivalente a F ( X ) en cada intervalo hasta la clase equivalente de transformación afín.

Considere la siguiente condición:

Condición 1: k I , gramo = C yo i pag ( F , k ) es continuo

¿Es esta condición una condición necesaria y suficiente para la continuidad absoluta de F en I ? ¿Se puede extender a funciones multidimensionales?


¿Está bien estudiado el morfismo "recorte"?


Esto está demostrando que gramo puede no ser continuo incluso si F es continuo gramo no es continua en X 1 medio: d ϵ y tal que | X 1 y | < ϵ pero | F ( X 1 ) F ( y ) | = d

Dejar F la función de Cantor. Dejar norte ser un número natural. En el norte -ésima etapa de la construcción del conjunto de Cantor, una colección disjunta [ X k , y k ] 1 < k < 2 norte de 2 norte subintervalos de I = [ 0 , 1 ] se han construido que cubren el conjunto de Cantor, cada uno de los cuales tiene una longitud ( 1 / 3 ) norte . La función de Cantor-Lebesgue es constante en cada uno de los intervalos que componen el complemento en [ 0 , 1 ] de esta colección de intervalos.

Tenemos k 2 norte ( y k X k ) = k < ( 2 / 3 ) norte mientras k 2 norte ( F ( y k ) F ( X k ) ) = 1

ϵ > 0 norte > 0 tal que k < ϵ .

Por definición, k = y k X k , y k 2 norte ( F ( y k ) F ( X k ) ) = gramo ( y k ) gramo ( X k ) .

Eso es : d = 1 ϵ y = y k tal que | X 1 y k | < ϵ pero | gramo ( X 1 ) gramo ( y k ) | = d . gramo no es continuo.


Es bien sabido que, en la definición de continuidad absoluta, la palabra "finito" puede ser reemplazada por "infinito numerable":

Una función F : I R es absolutamente continua en un intervalo I si por cada ϵ > 0 hay un d > 0 tal que siempre que una secuencia contable de subintervalos disjuntos por pares ( X k , y k ) de mi satisface

k | y k X k | < d
entonces
k | F ( y k ) F ( X k ) | < ϵ


Sabemos que la onda cuadrada se puede escribir como la suma infinita de formas de funciones seno: límite norte i = 1 norte h i ( X ) . Cada h i es continua pero el límite no lo es. Dejar H norte = i = 1 norte h i ( X ) . H norte debe ser continuo por inducción. Si H norte es de hecho continua en X = 0 entonces d ϵ y R tal que | y X | < ϵ implica | F ( y ) F ( X ) | < d . Sin embargo, ϵ > 0 norte > 0 tal que | y X | < ϵ y F ( y ) F ( 0 ) = 1 ¡contradición!


A Ramiro:

Supongamos F esta incrementando. Su respuesta parece implicar que, si F es continuo, entonces gramo es continuo, entonces d ϵ y norte tal que | y norte X 1 | < ϵ implica | gramo ( y norte ) gramo ( X 1 ) | < d ; sabemos k = i norte | y i X i | , y i norte | F ( y i ) F ( X i ) | = | gramo ( y norte ) gramo ( X 1 ) | , ENTONCES:

Tome cualquier colección de intervalo disjunto k tal que i norte | y i X i | < ϵ ,

esto implica k < ϵ ,

lo que implica | gramo k ( y norte ) gramo k ( X 1 ) | < d ,

lo que implica i norte | F ( y i ) F ( X i ) | < d . Entonces F es continuo absoluto!

No estoy seguro de que un solo gramo se requiere.

yo asumo eso I es un intervalo. Entonces, desde F es continua, la forma en que definiste C yo i pag ( F , k ) lo hace continuo, no importa si F es continuo absoluto o no.
@ramiro si F es la función de Cantor, es posible encontrar un conjunto de subintervalos que k es arbitrariamente pequeño pero i ( F ( y i ) F ( X i ) ) > 1 ; de este modo gramo no es continuo?
Si entiendo tu idea, estás definiendo gramo : [ X 1 , X 1 + k ] R por piezas (intervalos compactos). En el interior de cada intervalo gramo es continua (porque es una transformación afín de una pieza de F ). En el punto final entre dos intervalos cualesquiera, agrega constantes a F para asegurarse de que la pieza de la derecha coincida con la pieza de la izquierda, por lo que gramo también es continua en esos puntos. Entonces gramo es continua en todos los puntos de [ X 1 , X 1 + k ] .
Por cierto, creo que hay un pequeño error tipográfico cuando escribiste: "si X [ y k , y k + y k + 1 X k + 1 ] , entonces gramo ( X ) = F ( X y k + X k + 1 ) + C k + 1 , dónde C k + 1 = F ( y k ) F ( X k + 1 ) ". La expresión general para los intervalos (a partir del segundo) NO es [ y k , y k + y k + 1 X k + 1 ] . El segundo intervalo es [ y 1 , y 1 + y 2 X 2 ] , pero el tercero no es [ y 2 , y 2 + y 3 X 3 ] . El tercero debe ser [ y 1 + y 2 X 2 , y 1 + y 2 X 2 + y 3 X 3 ] . El tercer intervalo debe "comenzar" donde "termina" el segundo.
@Ramiro ¡Buen punto! recién actualizado
De acuerdo. Ahora los intervalos están bien. Si entiendo tu idea, para cada intervalo también hay un C norte para ser agregado a F , como el C 2 . ¿Bien?
@Ramiro Lo siento, mi error. Gracias por tomarme de la mano.
En "recorte", a medida que define, el número de intervalos siempre es finito , por lo que la prueba que publiqué (basada en inducción finita ) siempre funciona. Si F es la función de Cantor, cada "recorte" es continuo. Para aplicar el argumento que agregaste sobre la función de Cantor, debes usar límites y tener una secuencia contable infinita de intervalos. Entonces, tal argumento no se aplica al "recorte" como lo definió. Si fue este tipo de "argumento" lo que desea capturar en la definición de "recorte", entonces debe cambiar su definición de "recorte".
Para ayudar a aclarar algunos conceptos: 1. La inducción finita se aplica al caso de tener cualquier conjunto finito e incluso infinitos conjuntos contables. 2. La inducción no se aplica a la continuidad de la serie de Fourier, no porque la "repetición vaya al infinito", sino porque consideramos el límite puntual de una secuencia de funciones. En el ejemplo de la onda cuadrada, cada H norte es continuo! Lo que no es continuo es el límite puntual. límite norte H norte ( X ) . 3. El límite puntual de una sucesión de funciones continuas puede no ser una función continua.

Respuestas (1)

Suponer que F : I R es continuo, donde I es un intervalo. Tome cualquier conjunto finito k de subintervalos [ X 1 , y 1 ] , [ X 2 , y 2 ] , . . . , [ X norte , y norte ] I , dónde X 1 < y 1 X 2 < y 2 , . . . , < X norte y norte y k m ( [ X k , y k ] ) = k , es decir | k | = k . Entonces C yo i pag ( F , k ) es continuo

Prueba: Deja gramo = C yo i pag ( F , k ) . Probaremos por inducción que para cualquier k { 1 , . . . norte } , gramo es continua en [ X 1 , y k ] .

Para k = 1 , tenemos eso, gramo = F en [ X 1 , y 1 ] y y 1 = y 1 . Entonces gramo es continua en [ X 1 , y 1 ] .

Ahora supongamos que sabemos que gramo es continua en [ X 1 , y k ] , dónde 1 k < norte . Entonces tenemos eso

[ X 1 , y k + 1 ] = [ X 1 , y k ] [ y k , y k + 1 ]

Pero y k + 1 = X k + 1 + y k + 1 X k + 1 = y k + y k + 1 X k + 1 entonces tenemos

[ X 1 , y k + 1 ] = [ X 1 , y k ] [ y k , y k + y k + 1 X k + 1 ]

Ya que, por todo X [ y k , y k + y k + 1 X k + 1 ] , entonces gramo ( X ) = F ( X y k + X k + 1 ) + C k , y C k = gramo ( y k ) F ( X k + 1 ) . De este modo gramo es continua en [ y k , y k + y k + 1 X k + 1 ] y tenemos eso y k es

gramo | [ y k , y k + y k + 1 X k + 1 ] ( y k ) = F ( y k y k + X k + 1 ) + gramo ( y k ) F ( X k + 1 ) = gramo ( y k ) = gramo | [ X 1 , y k ] ( y k )

Entonces gramo es continua en [ X 1 , y k + 1 ] . por inducción, gramo es continua en [ X 1 , y norte ] . y desde y norte = X 1 + k , tenemos eso gramo es continua en [ X 1 , X 1 + k ] .

Conclusión: si F es continuo, entonces k I , gramo = C yo i pag ( F , k ) es continuo

Observación 1:

La expresión general para definir gramo es

Si ( X + y k ) [ y k , y k + y k + 1 X k + 1 ] , entonces gramo ( X + y k ) = F ( X + X k + 1 ) + C k , y C k = gramo ( y k ) F ( X k + 1 ) ,

que es equivalente a:

Si X [ y k , y k + y k + 1 X k + 1 ] , entonces gramo ( X ) = F ( X y k + X k + 1 ) + C k , y C k = gramo ( y k ) F ( X k + 1 ) .

Observación 2:

si por alguna razón realmente quieres que la expresión general defina gramo ser:

Si ( X + y k ) [ y k , y k + y k + 1 X k + 1 ] , entonces gramo ( X + y k ) = F ( X + y k ) + C k , y C k = gramo ( y k ) F ( y k ) .

la prueba anterior sigue siendo válida con cambios menores:

gramo | [ y k , y k + y k + 1 X k + 1 ] ( y k ) = F ( y k y k + y k ) + gramo ( y k ) F ( y k ) = gramo ( y k ) = gramo | [ X 1 , y k ] ( y k )

Observación 3 (respuestas a sus preguntas):

Como muestra la prueba anterior, su condición 1 es una condición necesaria para la continuidad de F en I . Por lo tanto, es una condición necesaria para la continuidad absoluta de F en I .

Además, como consecuencia de la prueba anterior, su condición 1 NO es una condición suficiente para la continuidad absoluta de F en I . De hecho, la condición 1 es verdadera para cualquier función continua, independientemente de si la función es absolutamente continua o no.

Finalmente, tenga en cuenta que al "recortar" F , un mecanismo clave fue que dos intervalos compactos "pegados" tienen solo un punto en común, y así, pudimos ajustar las piezas de F simplemente agregando una constante. En dimensiones superiores, este mecanismo no funciona. Por ejemplo, en general, dos rectángulos "pegados" tendrán un borde en común, no solo un punto.

Observación 4: (sobre la función de Cantor)

En el "recorte" como lo define, el número de intervalos siempre es finito , por lo que la prueba que publico (basada en la inducción FINITA) siempre funciona.

Si F es la función de Cantor, cada "recorte" es continuo.

Para aplicar el argumento que agregaste sobre la función de Cantor, debes usar límites y tener una secuencia contable infinita de intervalos. Entonces, el argumento que publicaste no es un contraejemplo de la prueba anterior. No se aplica al "recorte" como lo define.

Si fue este tipo de "argumento" lo que desea capturar en la definición de "recorte", entonces debe cambiar su definición de "recorte".

Observación 5:

Usted escribió: "Supongamos F esta incrementando. Su respuesta parece implicar que, si F es continuo, entonces gramo es continuo, entonces d ϵ y k tal que | y k X 1 | < ϵ implica | gramo ( y k ) gramo ( X 1 ) | < d ; sabemos k = | y i X i | , y i k | F ( y i ) F ( X i ) | = | gramo ( y k ) gramo ( X 1 ) | , ENTONCES

| y i X i | < ϵ implica | F ( y i ) F ( X i ) | < d . Entonces F es absolutamente continuo!"

Para que este argumento sea correcto, debe definir una sola función gramo tal que para cualquier secuencia finita de subintervalos i k | F ( y i ) F ( X i ) | = | gramo ( y k ) gramo ( X 1 ) | .

Sin embargo, según la definición de gramo , para cada secuencia finita de subintervalos, tenemos un continuo diferente " gramo ".

Observación 6:

Escribiste "Supongamos F esta incrementando. Su respuesta parece implicar que, si F es continuo, entonces gramo es continuo, entonces d ϵ y k tal que | y k X 1 | < ϵ implica | gramo ( y k ) gramo ( X 1 ) | < d ; sabemos k = | y i X i | , y i k | F ( y i ) F ( X i ) | = | gramo ( y k ) gramo ( X 1 ) | , ENTONCES:

Tome cualquier colección de intervalo disjunto k tal que | y i X i | < ϵ ,

esto implica k < ϵ ,

lo que implica | gramo k ( y k ) gramo k ( X 1 ) | < d ,

lo que implica | F ( y i ) F ( X i ) | < d . Entonces F es continuo absoluto! "

No. Este argumento no funciona. Dejame explicar.

  1. Primero supongamos que F es una función monótona (como la función de Cantor). Entonces gramo k es también una función monótona.

Vayamos paso a paso. La función gramo antes el "SO" ya es un gramo k para algunos k .

Por su continuidad, dada d > 0 hay un ϵ > 0 (que depende de d y en gramo k ) tal que, para cualquier colección de intervalo disjunto k tal que | y i X i | < ϵ , tenemos

| gramo k ( y k ) gramo k ( X 1 ) | < d

dónde y k corresponde al punto "más a la izquierda" de k (no de k ).

También tenemos eso:

( X i , y i ) k | F ( y i ) F ( X i ) | = | gramo k ( y k ) gramo k ( X 1 ) |
y
( X i , y i ) k | F ( y i ) F ( X i ) | = | gramo k ( y k ) gramo k ( X 1 ) |
pero ninguna de esas dos ecuaciones se combina con | gramo k ( y k ) gramo k ( X 1 ) | < d .

La falla en su argumento es el resultado de que, al escribir solo gramo , piensas en diferente gramo k funciones como si fueran una sola y la misma función. Lo mismo aplica y k , que en realidad depende de qué k estamos considerando.

  1. Si F no se supone que sea una función monótona, entonces incluso

( X i , y i ) k | F ( y i ) F ( X i ) | = | gramo k ( y k ) gramo k ( X 1 ) |

puede que no sea cierto para todos k , debido a los valores absolutos utilizados en la suma.

Gracias por escribir, votado. Actualicé en la pregunta principal el contraejemplo que gramo no siempre es continua si F es continuo Tenga en cuenta que el método de inducción no siempre funciona cuando la repetición llega al infinito ( math.stackexchange.com/questions/98093/… ); finito y contablemente infinito son algo equivalentes en el contexto de la continuidad.
La inducción no se aplica a la continuidad de la serie de Fourier, no porque la "repetición vaya al infinito", sino porque consideramos el límite puntual de una secuencia de funciones. Finito e infinito contable NO son en general equivalentes en el contexto de continuidad exactamente porque, en este contexto, cuando tenemos infinito contable necesitamos aplicar algún tipo de límite, que puede no preservar algunas propiedades. Por ejemplo, el límite puntual de una secuencia de funciones continuas puede no ser continuo.
En el ejemplo del cantor, mi norte siempre es finito. No importa cómo elijas un ϵ , siempre puedo encontrar un finito norte tal que k < ϵ y | gramo ( y k ) gramo ( X 1 ) | = 1 ; eso es, gramo no es continuo.
A pesar de nuestro desacuerdo, su respuesta sigue siendo muy, muy útil y constructiva para mí. Lo aprecio. ¿Podemos al menos estar de acuerdo en eso, si cambio la palabra "finito" a "contable" en la primera línea, gramo no es necesariamente continua cuando F es continuo?
@HighGPA En su ejemplo de Cantor, para cada ϵ > 0 , puedes encontrar un finito norte tal que k < ϵ y **el correspondiente** "g_\epsilon" satisfacen | gramo ϵ ( y k ) gramo ϵ ( X 1 ) | = 1 . Las funciones gramo ϵ no son una sola función y cada una de ellas es continua.
@HighGPA Estás concluyendo incorrectamente que gramo no es continuo, porque estás considerando todos los diferentes gramo ϵ funciones a una sola función gramo .
Creo que tienes razón, aunque no estoy tan seguro; mi "prueba" solo funciona para infinito contable no finito. Por la misma idea, creo que la respuesta aquí " math.stackexchange.com/questions/125345/… " tampoco es completamente correcta. ¿Cuáles son tus ideas?
¡Esperar! Traté de endurecer la prueba de que gramo no es continuo. d = 1 ϵ y = y k tal que | X 1 y k | < ϵ pero | gramo ( X 1 ) gramo ( y k ) | = d . gramo no es continuo. El detalle está en la pregunta. ¿Tendrías tiempo para comprobarlo? ¡Gracias!
@HighGPA En la respuesta que mencionaste, para cada d hay una secuencia finita diferente de subintervalos disjuntos por pares ( X k , y k ) de mi satisface
k | y k X k | < d
Entonces, si crea una función de "recorte" de cada una de esas secuencias, obtendrá varias "g", cada una de ellas continua (las de menor tamaño). d aumentando más agudo que los de mayor d , pero todos ellos continuos). Para construir una sola "g" (que no será continua) a partir de todas esas "g", deberá usar algún tipo de límite .
El límite está rigurosamente definido dentro del argumento épsilon-delta
@HighGPA No, vea el nuevo ** Comentario 5 ** en mi respuesta. Sigues pensando en "recortar" (el " gramo ") como una sola función. No lo es. Es una familia de funciones continuas.
Tengo que admitir que todavía estoy en confusión. Humildemente, creo que el argumento épsilon-delta ya implica algún tipo de límite y, por lo tanto, algún tipo de infinito, por lo que no puede hacer una inducción adecuada para probar la continuidad.
Para cada colección de intervalos k , si tenemos un gramo k ; si todos estos gramo k son continuos, entonces todos esos intervalos en F también debe satisfacer el argumento de continuidad absoluta épsilon-delta para F . Ver la respuesta actualizada
La inducción @HighGPA FINITE funciona perfectamente para demostrar la continuidad. Si cada operación preserva la continuidad, cualquier cantidad finita de tales operaciones también preservará la continuidad. Por favor, vea mi comentario 6 .
@HighGPA ¿Dejaste de votar las respuestas?
Realmente no entiendo tu comentario 6. | gramo k ( X i ) gramo k ( y i ) | = | F ( X i ) F ( y i ) | por definición tan obviamente, d > | gramo k ( X i ) gramo k ( y i ) | = | F ( X i ) F ( y i ) | ; por supuesto, podemos estar seguros de que la secuencia en consideración es k sí mismo. Sin embargo, admito que estoy confundido con la inducción.
@HighGPA Cambié el texto en mi Observación 6. Espero que sea más claro ahora.
Tu respuesta no merece un voto a favor; se merece miles